Вы находитесь на странице: 1из 24

Sensitivity Analysis

The optimal solution of a LPP is based on the


conditions that prevailed at the time the LP model
was formulated and solved. In the real world, the
decision environment rarely remains static and it
is essential to determine how the optimal solution
changes when the parameters of the model are
changed. That is what sensitivity analysis does. It
provides efficient computational techniques to
study the dynamic behaviour of the optimal
solution resulting from making changes in the
parameters of the model.
In studying the sensitivity analysis, we should be
familiar with the lingo that is being used in LPP
situations. A general LPP is of the form
Maximize (or Minimize)
n n
x c x c x c z + + + = ...
2 2 1 1
subject to the constraints
11 1 12 2 1 1
21 1 22 2 2 2
1 1 2 2
1 2
...
...
.
...
, ,..., 0
n n
n n
m m mn n m
n
a x a x a x b
a x a x a x b
a x a x a x b
x x x
+ + + =
+ + + =
+ + + =
>
The RHS constants of the constraints,
m
b b b ,..., ,
2 1
are referred to resources or availabilities of the
problem.
The objective coefficients,
n
c c c ,..., ,
2 1
are referred to as unit profits (or unit costs).
The decision variables,
n
x x x ,..., ,
2 1
are referred to as units of activities 1, 2, , n.
Dual Price of a constraint
This measure actually represents the unit worth
of a resource - that is it gives the contribution to
the objective function resulting from a unit
increase or decrease in the availability of a
resource. In terms of duality theory, the dual
price of a resource (=constraint) i, is precisely the
value of the optimal dual variable y
i
associated
with the constraint i. (Did you understand why it
is called dual price?). Other non-suggestive
names include shadow prices and simplex
multipliers.
Reduced cost of a variable x
j
(=activity j)

is defined as
Cost of consumed resources per unit of activity x
j
- profit per unit of activity x
j
= z
j
- c
j
Changes affecting feasibility
The feasibility of the current optimum solution
may be affected only if
(1) The RHS of the constraints are changed
OR
(2) A new constraint is added to the model.
In both cases, infeasibility occurs when at least
one element of the RHS of the optimal tableau
becomes negative that is, one or more of the
current basic variables become negative.
is changed to
First we consider the changes in the optimal
solution due to changes in the RHS b
i
. We note
that the optimal solution is given by

1
B

= B b x
where b is the old RHS and B is the basic
matrix. Remember B
-1
is found in the optimal
tableau below the entries which initially had an
identity submatrix.
The optimal value is given by
b B
1
B
C

= z
b
b
'
When , the corresponding
new solution and new objective value are got by
replacing
with .
b
'
b
Example 4.3-2 (Pages 135-136)
TOYCO assembles three types of toys: trains,
trucks and cars using three operations. The daily
limits on the available times for the three
operations are 430, 460, and 420 minutes
respectively. The profits per toy train, truck and
car are $3, $2, and $5 respectively. The assembly
times per train at the three operations are 1, 3, and
1 minute respectively. The corresponding times
per truck and per car are (2, 0, 4) and (1, 2, 0)
minutes respectively. A zero indicates that the
operation is not used.
Letting x
1
, x
2
, and x
3
represent the daily number of
units assembled of trains, trucks and cars, the LPP
model is:
Maximize
3 2 1
5 2 3 x x x z + + =
subject to
0 , ,
420 4
460 2 3
430 2
3 2 1
2 1
3 1
3 2 1
>
s +
s +
s + +
x x x
x x
x x
x x x
The optimal tableau is given in the next slide.
(Note: x
4
, x
5
, and x
6
are slack variables there.)
x6 0 2 0 0 -2 1 1 20
x3 0 3/2 0 1 0 1/2 0 230
z 1 4 0 0 1 2 0 1350
Basic z x1 x2 x3 x4 x5 x6 Sol
x2 0 -1/4 1 0 1/2 -1/4 0 100
This is the optimal tableau
Suppose that TOYCO wants to change
the capacities of the three operations
according to the following cases:
460 500 300 450
(a) 500 (b) 400 (c) 800 (d) 700
400 600 200 350
( ( ( (
( ( ( (
( ( ( (
( ( ( (

Use Sensitivity analysis to determine the optimal
solution in each case.
| |
(
(
(

= =

1 1 2
0 2 / 1 0
0 4 / 1 2 / 1
; 0 5 2
1
B C
B
Solution: We note
(b) New Solution is
=
(
(
(

6
3
2
x
x
x
(
(
(

(
(
(

=
'

600
400
500
1 1 2
0 2 / 1 0
0 4 / 1 2 / 1
1
b B
(
(
(

=
0
200
150
Since this is feasible, it is optimal and the new
optimal value =
(c) New Solution is
=
(
(
(

6
3
2
x
x
x
(
(
(

(
(
(

=
'

200
800
300
1 1 2
0 2 / 1 0
0 4 / 1 2 / 1
1
b B
(
(
(

=
400
400
50
1300 200 5 150 2 0 3 = + +
This is not feasible. So we apply dual simplex
method to restore feasibility. We note
1900 400 5 50 2 0 3 = + +
new z =
x6 0 2 0 0 -2 1 1 400
x3 0 3/2 0 1 0 1/2 0 400
z 1 4 0 0 1 2 0 1900
Basic z x1 x2 x3 x4 x5 x6 Sol
x2 0 -1/4 1 0 1/2 -1/4 0 -50
x6 0 1 4 0 0 0 1 200
x5 0 1 -4 0 -2 1 0 200
z 1 2 8 0 5 0 0 1500
x3 0 1 2 1 1 0 0 300
This is the new optimal tableau.
Feasibility Range of the Elements of the RHS
Another way of looking at the effect of changing
the availabilities of the resources, b
i
, is to
determine the range for which the current
solution remains feasible.
For example if, in the TOYCO model, b
2
is
changed to b
2
+D
2
= 460+D
2
, we want to find the
range of D
2
so that the current solution remains
optimal.
When b
2
is changed to b
2
+D
2
= 460+D
2
, the new
solution is
=
(
(
(

6
3
2
x
x
x
(
(
(

+
(
(
(

=
'

420
460
430
1 1 2
0 2 / 1 0
0 4 / 1 2 / 1
2
1
D b B
(
(
(

+
(
(
(

=
2
2
2
2 / 1
4 / 1
20
230
100
D
D
D
(current optimal
solution + D
2
times the
2
nd
column of B
-1
.)

(
(
(

+
+

=
2
2
2
20
2 / 1 230
4 / 1 100
D
D
D
is feasible if
0 20
0 2 / 1 230
0 4 / 1 100
2
2
2
> +
> +
>
D
D
D
20
460
400
2
2
2
>
>
s
D or
D or
D or
400 20
2
s s D Or
Thus current solution remains optimal if RHS of
the 2
nd
constraint lies between 440 and 860 (the
other RHSs being the same).
Problem 5 Problem Set 4.5B Page 151
HiDec produces two models of electronic gadgets
that use resistors, capacitors, and chips. The
following table summarizes the data of the situation:
Unit Resources Requirements
Resource Model 1 Model2 Maximum Availability
(units) (units) (units)
Resistor 2 3 1200
Capacitor 2 1 1000
Chips 0 4 800
Unit Profit($) 3 4
Let x
1
, x
2
be the amounts produced of Models 1 and
2 respectively. Then the above model becomes the
LPP
Maximize
2 1
4 3 x x z + =
subject to
0 ,
800 4
1000 2
1200 3 2
2 1
2
2 1
2 1
>
s
s +
s +
x x
x
x x
x x
(Resistors)
(Capacitors)
(Chips)
Taking the slack variables as s
1
, s
2
, s
3
, the optimal
tableau is:
x2 0 0 1 1/2 -1/2 0 100
s3 0 0 0 - 2 2 1 400
z 1 0 0 5/4 1/4 0 1750
Basic z x1 x2 s1 s2 s3 Sol
x1 0 1 0 -1/4 3/4 0 450
This is the optimal tableau
(a) Determine the status of each resource
Answer: Since s
1
= 0 = s
2
, the resistor and the
capacitor resources are scarce.
Since s
3
> 0, the chips resource is abundant.
(b) In terms of the optimal profit, determine the
worth of one resistor, one capacitor and one chip.
Answer: They are respectively y
1
, y
2
, y
3
the
dual optimal solution and hence are
5/4, 1/4, 0 respectively.
(c) Determine the range of applicability of the dual
prices for each resource.
Resistor: If D
1
is the increase in the resource 1,
the new optimal solution is given by
=
(
(
(

2
3
1
x
s
x
(
(
(

+
(
(
(

=
'

800
1000
1200
0 2 / 1 2 / 1
1 2 2
0 4 / 3 4 / 1
1
1
D
b B
(
(
(

+
(
(
(

=
1
1
1
2 / 1
2
4 / 1
100
400
450
D
D
D
> 0 gives
200 200
1
s s D
Similar calculations show that, for a change D
2
in
the capacitor, the range of feasibility is given by
200 200
2
s s D
And for a change D
3
in the chips, the range of
feasibility is given by 400
3
> D
(d) If the available number of resistors is increased to
1300 units, find the new optimum solution.
The new solution is:
=
(
(
(

2
3
1
x
s
x 450 25
400 200
100 50

( (
( (
+
( (
( (

(
(
(

=
150
200
425 This is feasible
and hence
optimal.
And z = 1875.
Yes, as D
1
= 100
(g) A new contractor is offering to sell HiDec
additional resistors at 40 cents each but only if
HiDec would purchase at least 500 units. Should
HiDec accept the offer?

Вам также может понравиться